Kugelwelle als Summe ebener Wellen

Wie können wir diese Berechnung durchführen?

R 3 e ich k ' R R e ich k 1 X + k 2 j + k 3 z D X D j D z
Wo R = X 2 + j 2 + z 2 ? Ich denke, wir müssen Distributionen verwenden.

Physikalisch ist es äquivalent, Wellenvektoren zu finden k Verteilung und eine Kugelwelle als Summe ebener Wellen zu schreiben. Ich kenne die Formel für das inverse Problem: Schreibe eine ebene Welle als Summe von Kugelwellen. Die Lösung in diesem Fall ist eine Reihe von sphärischen Harmonischen und sphärischen Bessel-Funktionen.

Dies ist die 3D-Fourier-Transformation von e ich k ' R R ? Wenn ja, sollte die Formel sein v e ich k ' R R e ich k R D 3 R
Es ist dasselbe: wie in 1D: Sie können Fourier-Trasform als definieren F ( X ) e ± ich k X D X , und die inverse Trasform ist 1 2 π F ~ ( k ) e ich k X D k ;-)
Ja natürlich, aber die Definition des Wellenvektors wird in die entgegengesetzte Richtung gehen.
Diese Frage mag eine physikalische Motivation haben, aber ich denke, sie ist rein mathematischer Natur. Sie sollten wahrscheinlich bei math.SE nach der Antwort fragen.
@Marek: Ich glaube, ich stimme zu ... Ich meine, der Titel deutet auf eine physikalische Frage hin, aber im Wesentlichen geht es nur darum, wie man ein Integral macht. Ich stimme dafür, es zu schließen (aber nur, weil vier andere Leute zustimmen müssen, bevor die Frage tatsächlich geschlossen wird - ich würde es nicht bequem finden, dies einseitig zu schließen, wenn ich die Macht dazu hätte).
@David: Es tut mir leid, aber ich wusste nicht, ob die Frage mit der "rigorosen Methode" lösbar war oder ob dies sehr kompliziert sein könnte, also habe ich hier gefragt, weil ich dachte, dass ein Physiker wusste, dass dieses einfache Problem sogar wahrscheinlicher war Wenn die Lösung nicht "rigoros" wäre ... Ich habe bereits Fragen wie diese gepostet ... Es tut mir leid, wenn Sie möchten, lösche ich diese Frage ...
@ David: Richtig, ich würde es auch nicht bequem finden, es einfach alleine zu schließen. @Boy: kein Grund zur Entschuldigung, das Schließen einer Frage ist keine große Sache. Es geht wirklich mehr darum, Grenzen zu setzen, was auf dieser Seite gefragt werden sollte und was nicht, damit die Leute es in Zukunft wissen.
Ich weiß, dass ich in der Minderheit bin, aber ich mag es, diese "mathematischen" Fragen hier zu sehen. Physiker haben eine andere Mathematikkultur als Mathematiker und erhalten eher nützliche Antworten von anderen Physikern.
@nibot: Obwohl ich (vorerst) anderer Meinung bin, ist das ein ausgezeichneter Punkt. Und @Boy Simone: Dem letzten Kommentar von Marek stimme ich voll und ganz zu, kein Grund zur Entschuldigung. Sie haben nichts falsch gemacht, indem Sie diese Frage gestellt haben (und tatsächlich helfen Sie uns, den Umfang dieser Website zu definieren).
@David, @Marek, ich stimme @nibot zu. Wir haben auf dieser Website ein Tag namens Mathematische Physik, und ich denke, diese Art von Frage verdient dieses Tag und gehört daher zu dieser Website. Manchmal brauchen wir keine allzu strenge oder komplizierte mathematische Antwort auf eine mathematische Frage, und dann kann die Sichtweise eines Physikers hilfreich sein. Ich hoffe also, dass diese Art von Frage in Zukunft nicht geschlossen wird.

Antworten (2)

Aus Ihrer Beschreibung geht hervor, dass Sie die Fourier-Transformation von finden möchten

F ( R ) = e ich k ' R R ,
und die Welle kann aus der linearen Überlagerung von ebenen Wellen, identifiziert durch k, wiedergewonnen werden
F ( R ) = 1 ( 2 π ) 3 / 2 F [ F ] ( k ) e ich k R D 3 k .

Die sphärische Welle hat sphärische Symmetrie, also sollten Sie die Integration in sphärischen Koordinaten statt in kartesischen durchführen. WLOG, nehmen wir also an, dass k entlang der z- Achse verläuft

F [ F ] ( k z ^ ) = 1 ( 2 π ) 3 / 2 e ich k ' R R e ich k R D 3 R = 1 ( 2 π ) 3 / 2 e ich k ' R R e ich k R cos θ R 2 Sünde θ D R D θ D ϕ = 1 ( 2 π ) 1 / 2 0 ( R e ich k ' R 0 π e ich k R cos θ Sünde θ D θ ) D R = 1 ( 2 π ) 1 / 2 0 R e ich k ' R 2 Sünde k R k R D R = 1 k 2 π 0 e ich k ' R Sünde k R D R = 2 π 1 k 2 k ' 2

Danke! Und wie hast du den letzten Durchgang gemacht? 1 / k 0 + ich N F T j e ich k ' R S ich N k R D R = 1 k 2 + k ' 2
@Junge: Was meinst du?
Ich denke, er möchte wissen, wie Ihr letzter Schritt der Berechnung durchgeführt wird, dh das Integral über e ich k ' R Sünde k R .
@Junge: Ändern Sünde k R Zu ( e ich k R e ich k R ) / 2 ich . Dann beachte das 0 e ich K R D R = ich / K (Wenn wir Konvergenzsachen ignorieren. Eigentlich ist die Antwort nicht richtig, wann k ' hat keinen positiven Imaginärteil, da das Integral divergiert.)
Ja, dann bin ich auch zu dieser Überlegung gekommen. Mein Zweifel ist nur, dass ein rein reelles k keine Absorption bedeutet, aber ich denke, dass Sie eine rein sphärische Welle nicht mit ebenen Wellen erklären können, ist sehr seltsam, weil das Gegenteil möglich ist ... Vielleicht gibt es einen Weg, Konvergenz zu erreichen von dieses Integral im Sinne der Verteilung ... wie e ich k X = 2 π δ ( k ) ...Ich poste die Frage auch in den mathematischen Abschnitt, weil das wirklich rein mathematisch ist ... Danke :-)
PS, Yeha, ich denke, dass es dasselbe ist: Es ist ähnlich wie die Antitrasform in der Lösung der Poisson-Gleichung, wenn Sie die grüne Funktion finden ...! :-)
Beim letzten Integral muss man durch Verschieben für die Konvergenz des Integrals sorgen k ' leicht in den Komplex: ( k ' + ich ϵ ) , ϵ > 0 . Für eine ankommende Welle e ich k ' R / R man muss wählen ( k ' ich ϵ ) , ϵ > 0 . Andernfalls hätten ein- und ausgehende Kugelwellen die gleiche Fourier-Transformation.
Bei dieser Antwort fehlt der Imaginärteil. Siehe meine Antwort für die Lösung.
0 + e ich k ' R Sünde k R D R = 1 2 ich 0 + e ich ( k ' + k ) R D R 1 2 ich 0 + e ich ( k ' k ) R D R . 0 + e ich ω T D T = + θ ( T ) e ich ω T D T = 1 ich ω + π δ ( ω ) . Siehe en.wikipedia.org/wiki/… (Punkt 313, dritte Version der Fourier-Transformation), hier θ ( T ) ist die Heaviside-Funktion. 1 2 ich 0 + e ich ( k ' + k ) R D R = 1 2 ich ( 1 ich ( k ' + k ) + π δ ( k ' + k ) ) = 1 2 ( k ' + k ) ich π 2 δ ( k ' + k ) .
1 2 ich 0 + e ich ( k ' k ) R D R = 1 2 ich ( 1 ich ( k ' k ) + π δ ( k ' k ) ) = 1 2 ( k ' k ) ich π 2 δ ( k ' k ) .
0 + e ich k ' R Sünde k R D R = 1 2 ( k ' + k ) 1 2 ( k ' k ) ich π 2 δ ( k ' + k ) + ich π 2 δ ( k ' k ) = = k k 2 k ' 2 + ich π 2 ( δ ( k k ' ) δ ( k + k ' ) )
Die endgültige Antwort lautet: F [ F ] ( k z ^ ) = 2 π 1 k 2 k ' 2 + ich k π 2 ( δ ( k k ' ) δ ( k + k ' ) ) .

Die Antwort von kennytm ist nur teilweise richtig. Es findet alle Werte des Fourier-Bildes – wo sie existieren. Aber das vollständige Fourier-Bild einer Kugelwelle ist keine Funktion, sondern eine Verteilung .

Betrachten wir eine stehende Welle, die anhand von beschrieben wird 0 sphärische Bessel-Funktion ter Ordnung (der Imaginärteil der OP-Funktion):

G ( R ) = k ' J 0 ( k ' R ) = Sünde ( k ' R ) R .

Wir können seine Fourier-Transformation ähnlich dem Ansatz in der Antwort von kennytm finden , jedoch mit einer besonderen Behandlung des endgültigen Integrals:

ICH = 0 Sünde ( k ' R ) Sünde k R D R .

Dieses Integral (bis zur multiplikativen Konstante) ist die Sinustransformation von Sünde ( k ' R ) , was gleich ist

ICH = δ ( k k ' ) δ ( k + k ' ) ,

Wo δ ist das Dirac-Delta .

In ähnlicher Weise können wir feststellen, dass die Fourier-Transformation der zweiten Kugelwelle – diejenige mit 0 Sphärische Neumann-Funktion ter Ordnung (der Realteil der OP-Funktion):

H ( R ) = k ' j 0 ( k ' R ) = cos ( k ' R ) R .

Die Fourier-Transformation von diesem würde (bis zu einem konstanten Multiplikator) auf die Sinustransformation von reduzieren cos ( k ' R ) , und wir erhalten endlich die gleiche Fourier-Transformation wie in kennytms Antwort:

F [ H ] ( k z ^ ) = 2 π 1 k 2 k ' 2 .

Jetzt können wir die vollständige Fourier-Transformation der im OP angegebenen Laufwelle kompilieren:

F ( R ) = e ich k ' R R .

Es ist die Kombination der beiden oben gefundenen Ergebnisse:

F [ F ] ( k z ^ ) = 2 π 1 k 2 k ' 2 + ich k π 2 ( δ ( k k ' ) δ ( k + k ' ) ) .

Ich denke, die Antwort lautet:
F [ F ] ( k z ^ ) = 2 π 1 k 2 k ' 2 + ich k π 2 ( δ ( k k ' ) δ ( k + k ' ) ) .
Ich habe die Kommentare mit Berechnungen zur Antwort von kennytm hinzugefügt.
@rtmd du hast recht, ich habe die Antwort jetzt korrigiert.